LSAT 90 – Section 4 – Question 08

You need a full course to see this video. Enroll now and get started in less than a minute.

Target time: 0:58

This is question data from the 7Sage LSAT Scorer. You can score your LSATs, track your results, and analyze your performance with pretty charts and vital statistics - all with a Free Account ← sign up in less than 10 seconds

Question
QuickView
Type Tags Answer
Choices
Curve Question
Difficulty
Psg/Game/S
Difficulty
Explanation
PT90 S4 Q08
+LR
Main conclusion or main point +MC
A
0%
144
B
5%
154
C
77%
162
D
13%
153
E
4%
152
138
147
157
+Medium 148.293 +SubsectionMedium

This is a Main Conclusion question.

This question is pretty cookie-cutter. Structurally, we are given 1) a phenomenon, 2) other people's hypothesis, and 3) an alternate hypothesis. What makes this question challenging despite this cookie-cutter structure is the presence of two really attractive answer choices.

Let's first look at the stimulus. The climatologist says the waters off the Pacific coast of North America have warmed about 4 degrees over the last 15 years. This is the phenomenon. And then she tells us other people’s hypothesis first (you can also call it an explanation or a conclusion, but I will call it a hypothesis here for simplicity). Some scientists claim that this trend of warming is a symptom of a more general global warming caused by human-generated air pollution.

And our author, predictably, says this conclusion from other scientists is far from justified. That is our author's conclusion. And why should we believe this? Because of her premise. Because it is known that there are many natural cycles of ocean temperature changes that last 60 years or more. Her premise is an alternate hypothesis for the very same phenomenon. The other scientists’ claims are far from justified because they have not considered this alternate hypothesis.

That is the stimulus, but the answer choices get kind of tricky.

Let’s start with the Correct Answer Choice (C), which says that the conclusion that the warming of the waters off the Pacific coast of North America is a symptom of a more general, global warming caused by human-generated air pollution is far from justified. This is a perfect fully fleshed out version of “this conclusion is far from justified,” where “this conclusion” refers to some scientists’ claims that the warming of waters is caused by a general global warming.

Contrast (C) with Answer Choices (B) and (D), two really attractive choices. I think both (B) and (D) would be correct in Reading Comprehension Inference questions that ask what the author would agree with. I think she would agree that the warming of the waters is not a symptom of global warming. I think she would also agree that the warming of the water may be the result of natural cycles, and this is probably even better supported since it uses softer language. But neither is the main conclusion.

Let’s look at (B) first. (B) is a cookie-cutter wrong answer choice. There are many Flaw questions that explicitly test this error made in (B).

Say a patient shows up with symptoms of high fever, and her first doctor says this is because of the common cold. And then a second doctor says, "This conclusion is far from justified." Why? Her premise to support this conclusion is some alternative hypothesis that might also explain the phenomenon, like the flu.

If this were a Flaw question, you have to be careful in how you describe what the second doctor is doing. She is questioning the first doctor's reasoning for failing to consider an alternative hypothesis. Put differently, the second doctor is merely questioning the support structure of someone else's argument. That is not the same thing as concluding that the patient does not have a cold, or that the patient indeed has the flu.

If this were an Inference from the author's perspective question, then you may well say that the second doctor thinks that the patient doesn't have a cold. But just because she revealed the weakness in someone's reasoning does not mean that she has shown that their conclusion is false. In order to do that, she still has to come up with her own argument to support what we think she believes: that the patient doesn't have a cold.

(B) is making this mistake by saying the warming of the waters off the Pacific coast is not a symptom of a more general global warming. That is not the author's conclusion, even if we think the author would agree. The author's conclusion is simply that people who claim this have not done a good job of supporting this claim.

And (D) makes a similar mistake. (D) says the warming of the waters off the coast may be the result of a natural cycle of ocean temperature changes. The climatologist would probably agree with this, but it does not matter because the question is asking us to identify the main conclusion.

Answer Choice (E) is similar. (E) says if the warming of the waters off the Pacific coast of North America is due to natural cycles of temperature change, then it is not a symptom of global warming. Like (B) and (D), I think the author would agree with (E). (E) could be some sort of an underlying principle that says you do not need to consider a hypothesis if a phenomenon can be explained by some other hypothesis. But that does not mean it is the main conclusion of the argument.

Answer Choice (A) says some scientists have found evidence that the waters off the Pacific coast have grown warmer over the past 15 years. I do not even know if this is true. The phenomenon of waters warming is declared as a fact without any evidence offered, and this is certainly not the main conclusion.

Take PrepTest

Review Results

Leave a Reply